LSAT and Law School Admissions Forum

Get expert LSAT preparation and law school admissions advice from PowerScore Test Preparation.

 ars316
  • Posts: 1
  • Joined: Aug 13, 2014
|
#15891
I am currently attempting to complete the lesson 3 HW and am on page 3-31. So far, I have gotten both questions 1 and 2 incorrect and I am not sure why my answers are wrong. The explanations in the book are not thorough enough in explaining why my answers are wrong. My initial setup of the problem is correct but my not laws aren't matching up with the ones in the book. I look forward to hearing from someone shortly.
Thanks,
Arielle
User avatar
 KelseyWoods
PowerScore Staff
  • PowerScore Staff
  • Posts: 1079
  • Joined: Jun 26, 2013
|
#15908
Hi Arielle,

Thanks for your question and welcome to the forum! It would help if you could be a little more specific about where exactly you get tripped up on these setups and let us know your thought process. But I'll try to explain the Not Laws in drills #1 & #2:

For drill #1, we know that P is assigned a lower numbered lane than K and K is assigned a lower numbered lane than J. That gives us the P > K > J sequence. Since J has to have at least two variables before him (P & K), that means J cannot go in lanes 1 or 2. Since P has to have at least two variables after him (K & J), that means P cannot go in lanes 5 or 6. Since K has to have at least one variable before her (P) and at least one variable after her (J), K can go neither 1st nor last (lane 6).

For drill #2, we know that A is two places before D (A __ D). This means that D must have at least two variables (A and __) before it so D cannot go 1st or 2nd. Similarly, A must have at least two variables after it (__ & D) so it can go neither 4th nor 5th. Since the A__D block can only go in three locations (1&3, 2&4, 3&5), you can draw out templates to show each of those three options, as they did at the bottom of page 3-106. In each of these options, you can narrow down where E goes since E cannot go next to D. From these options, we see that no matter where the A__D block is, E can never go 3rd or 4th.

Hope this helps!

Best,
Kelsey

Get the most out of your LSAT Prep Plus subscription.

Analyze and track your performance with our Testing and Analytics Package.